§1. Mệnh đề

Bạn chưa đăng nhập. Vui lòng đăng nhập để hỏi bài
Anh Trâm

Cho 0<a,b,c<1.CMR: có ít nhất 1 trong các bdt sau là đúng:

\(a\left(1-b\right)\le\frac{1}{4};b\left(1-c\right)\le\frac{1}{4};c\left(1-a\right)\le\frac{1}{4}\)

Đinh Đức Hùng
14 tháng 9 2019 lúc 13:23

§1. Mệnh đề


Các câu hỏi tương tự
nga thanh
Xem chi tiết
Nguyễn Thị Bình Yên
Xem chi tiết
Nguyễn Thanh
Xem chi tiết
Thị Mỹ Hạnh Võ
Xem chi tiết
Nhi Lê Nguyễn Bảo
Xem chi tiết
Nguyễn Thanh
Xem chi tiết
Ngọc Minh Đinh
Xem chi tiết
Thảo Thu
Xem chi tiết
Lê Hồng Ngọc
Xem chi tiết